Théorie > Théorie des nombres > Divers résultats et méthodes

Exemples d'application

Nous donnons ici deux applications concrètes du LTE. La première est plus ou moins directe (on peut penser au LTE dès qu'on lit le problème), alors que la deuxième est un peu plus cachée.

Problème (IMO Shortlist 1991)
Trouver le plus grand degré $k$ de $1991$ tel que $1991^k$ divise $1990^{1991^{1992}} + 1992^{1991^{1990}}$.

Solution
Puisque $1991$ divise $1990+1992$, il est très tentant d'utiliser le LTE. Mais cela ne va quand même pas être aussi simple : les exposants de $1990$ et $1992$ ne sont pas les mêmes et, en plus, $1991$ n'est pas premier ! On a en effet $1991 = 11 \cdot 181$. Ce deuxième obstacle peut facilement être passé : nous allons essayer d'utiliser le LTE avec successivement $p = 11$ et $p = 181$. Pour ce qui est des exposants qui ne sont pas les mêmes, on peut en fait simplement constater que :
$$1990^{1991^{1992}} + 1992^{1991^{1990}} = \left(1990^{1991^2}\right)^{1991^{1990}} + 1992^{1991^{1990}}.$$ On a donc bien une expression de la forme $a^n+b^n$, avec $a = 1990^{1991^2}$, $b = 1992$ et $n = 1991^{1990}$. Prenons $p = 11$ et vérifions les hypothèses du LTE (deuxième forme). On a $n$ qui est bien impair, et $p$ qui est bien différent de $2$. Il suffit donc d'avoir $p$ qui divise $1990^{1991^2} + 1992$ pour pouvoir appliquer le lemme. Cela peut se voir de plusieurs manières. La plus simple est peut-être de regarder l'expression modulo $p$ : puisque $1991 \equiv 0 \bmod p$, on a $1990 \equiv -1$ et $1992 \equiv 1$, d'où $1990^{1991^2} + 1992 \equiv (-1)^{1991^2} + 1 \equiv 0 \bmod p$. Les hypothèses du LTE sont donc vérifiées et celui-ci donne
$$v_p\left(\left(1990^{1991^2}\right)^{1991^{1990}} + 1992^{1991^{1990}}\right) = v_p\left(1990^{1991^2} + 1992\right) + v_p(1991^{1990}).$$ On a $v_p(1991^{1990}) = 1990$, et il reste à évaluer $v_p\left(1990^{1991^2} + 1992\right)$. Pour cela, on peut utiliser le résultat général suivant (celui-ci est très important et très facile à la fois : la démonstration est laissée au lecteur) :

Lemme
Soit $p$ un nombre premier et $x, y$ deux entiers. On a
$$v_p(x+y) \geq \min(v_p(x), v_p(y)) \text{ avec égalité lorsque $v_p(x) \neq v_p(y)$}.$$

Suite de la solution
Dans notre contexte, en prenant $x = 1990^{1991^2}+1$ et $y = 1991$, on obtient $v_p(x) = 3$ (à nouveau à l'aide du LTE !) et $v_p(y) = 1$, donc $v_p(x+y) = 1$ (car $3 \neq 1$).

Au final, nous avons prouvé que la valuation $p$-adique de l'expression de départ est égale à $1990 + 1 = 1991$. Tout notre raisonnement est valide pour $p = 11$ mais aussi pour $p = 181$, donc on en déduit que la réponse au problème est $k = 1991$.

Voici un autre problème :

Problème (IMO 1990, Problème 3)
Trouver tous les entiers $n > 1$ tels que l'expression
$$\frac{2^n+1}{n^2}$$ est un entier.

Si l'on souhaite appliquer le LTE à l'expression $2^n+1$, alors on ne peut prendre pour $p$ qu'un diviseur de $2+1$, c'est-à-dire $3$. Pour cette raison nous allons d'abord prouver le petit lemme suivant :

Lemme
Si $n^2$ divise $2^n+1$, alors $n$ est divisible par $3$.

Démonstration du lemme
Considérons $n$ tel que $n^2$ divise $2^n+1$. Notons $p$ le plus petit facteur premier de $n$ : clairement $p \neq 2$. On a $2^n +1 \equiv 0 \bmod p$, donc $2^n \equiv -1 \bmod p$ et $2^{2n} \equiv 1 \bmod p$. Or, on a aussi $2^{p-1} \equiv 1 \bmod p$ par le théorème de Fermat. On en déduit que l'ordre multiplicatif $o_p(2)$ de $2$ modulo $p$ doit diviser à la fois $2n$ et $p-1$. Mais $n$ et $p-1$ sont premiers entre eux, puisque $p$ est le plus petit facteur premier de $n$. Donc $o_p(2) = 2$ (il ne peut clairement pas être égal à $1$) et $2^2 \equiv 1 \bmod p$, d'où $p = 3$. Cela prouve que $n$ est divisible par $3$.

On peut maintenant utiliser ce lemme pour résoudre le problème.

Solution du problème
Par le lemme, $n$ est divisible par $3$. Nous appliquons alors le LTE (deuxième forme) avec $p = 3$. On a $v_3(2^n+1) = v_3(2+1) + v_3(n) = 1+v_3(n)$ (les hypothèses sont bien vérifiées). D'autre part, $v_3(n^2) = 2 \cdot v_3(n)$. Pour que $n^2$ divise $2^n+1$, il faut donc que $2 v_3(n) \leq 1+v_3(n)$. On en déduit que $v_3(n) =1$ et donc que $n$ est de la forme $n = 3 m$ avec $m$ non multiple de $3$.

Nous faisons à présent le même argument que dans notre lemme. Supposons $m > 1$ et notons $q$ le plus petit facteur premier de $m$. On a $2^{3m} \equiv -1 \bmod q$ et donc $2^{6m} \equiv 1 \bmod q$. D'autre part, par Fermat on sait que $2^{q-1} \equiv 1 \bmod q$. Donc l'ordre multiplicatif $o_q(2)$ de $2$ modulo $q$ divise $6m$ et $q-1$. Comme $m$ et $q-1$ n'ont aucun facteur commun, $o_q(2)$ divise $6$. Ainsi, $2^6 \equiv 1 \bmod q$. Donc $q$ divise $2^6-1 = 63$ et ainsi $q = 7$ (car $q \neq 3$).

Cela étant, une expression de la forme $2^n+1$ n'est jamais divisible par $7$ ! En effet, les puissances de $2$ modulo $7$ donnent successivement $1, 2, 4, 1, 2, 4, \ldots$, et donc $2^n+1$ n'est jamais divisible par $7$. Nous avons donc une contradiction : $q$ ne peut pas être égal à $7$. Cela signifie que le nombre $m$ était en fait égal à $1$, d'où $n = 3$. Le seul entier $n$ vérifiant l'énoncé est donc $n = 3$ (qui fonctionne bien : $3^2 = 9$ divise $2^3+1 = 9$).